7
$\begingroup$

The following (very simply looking!) problem occurs in regularization of the harmonic series which can be formally thought of as the limit as $q\to1$, $|q|<1$, of $$ h(q):=(1-q)\sum_{n=1}^\infty\frac{q^n}{1-q^n}. $$ I can show (with some effort) that $$ h(q)=-\log(1-q)+f(q) \qquad\text{as}\quad q\to1, \ |q|<1, $$ where $f(q)$ is a bounded function (hint: consider both $h(q)$ and $h(q^2)$ as $q\to1$). The question is whether the function $f(q)$ has a limit as $q\to1$ or not; in other words, whether $$ h(q)=-\log(1-q)+c+o(1) \qquad\text{as}\quad q\to1, \ |q|<1. $$ Then, of course, I am very much interested in the constant $c$. A straightforward computer experiment is not helpful.

$\endgroup$

2 Answers 2

9
$\begingroup$

Andrew is right, the following limit seems to be what you are looking for $$\lim_{q\uparrow 1}\left(\log(1-q)-\log q \sum_{n\geq 0}\frac{q^{n+1}}{1-q^{n+1}}\right)=\gamma$$ See , for example theorem 1 in "Summations for Basic Hypergeometric Series Involving a $q$-Analogue of the Digamma Function" by C. Krattenthaler and H.M. Srivastava. (Though there should be a more canonical source for this somewhere.)

$\endgroup$
1
  • $\begingroup$ Gjergji, this is extremely helpful! (And because your link does not provide a correct link to the paper itself, I add it here: mat.univie.ac.at/~kratt/artikel/digamma.html). I am very pleased to learn that it is actually done by one of my collaborators... $\endgroup$ Jul 4, 2011 at 15:00
10
$\begingroup$

The function $h(q)$ is equal to $$\frac{(1-q) \left(\log \left(\frac{1}{1-q}\right)-\psi _q(1)\right)}{\log \left(\frac{1}{q}\right)},$$ where $\psi _q(z)$ is the $q$-digamma function. According to Mathematica $с$ is equal to the Euler's constant $\gamma$.

$\endgroup$
1
  • $\begingroup$ Andrew, I definitely vote up, but can only accept Gjergji's answer as it also provides the required details of the evaluation. Mathematica is famous for hiding the tools it actually uses. But I am very thankful to you as well. $\endgroup$ Jul 4, 2011 at 15:03

Your Answer

By clicking “Post Your Answer”, you agree to our terms of service and acknowledge you have read our privacy policy.

Not the answer you're looking for? Browse other questions tagged or ask your own question.